You are on page 1of 7

Math Problem Solutions September 8, 2019

Problem 1
Let f(x) = ln( 1−x
1+x )
4x
f”(x) = - t2 (where t = 1 - x2 ) =⇒ f”(x) ≤ 0 for x ∈ (0,1).
P5 P5
f (ai ) i=1 (ai )
=⇒ f is concave over the interval (0,1). =⇒ i=1
5 ≤ f( 5 ) = f( 15 )
P5 1−ai
i=1 ln( )
=⇒ 1+ai
≤ ln( 32 ).
Q5 51−ai
=⇒ i=1 ( 1+ai ) ≤ ( 23 )5

Problem 2
Claim: There is no such set S
y 2 y 2
Proof: : Suppose to the contrary ∃ a set S. =⇒ x ≤ 2m−1 or x ≥ 2m+1 for all m,x,y ∈ N

16 l1
l2

14

12

10

−6 −4 −2 2 4 6 8 10 12 14 16 18 20 22 24

−2

l3

−4

Fix m = m0 ; so now the no of integer pairs satisfying the conditions is the number of lattice
points in the area between l1 : xy = 2m20 −1 and l2 : xy = 2m20 +1 . Now it should also hold for
m = m0 + 1, conditions now become :
y 2 y 2
x ≤ 2m0 +1 or x ≥ 2m0 +3 . So now number of integers is the number lattice points in the area
between l2 : x = 2m0 +1 and l3 : xy = 2m20 +3 . Clearly these 2 areas are disjoint [since lattice
y 2

points on the lines l1 , l2 , l3 are not considered].


=⇒ S is empty.
=⇒ contradiction. This proves our claim. 

Solutions by Aahan Chatterjee Page 1


Math Problem Solutions September 8, 2019

Problem 3

O C

E
B

IA

let 0 be the circumcenter of 4ABC.


Claim: O ∈ IA H
Proof: : Invert wrt. the A-Excircle of 4ABC. [The inverse of a point X will be denoted by
X1 .]
Note that B1 ∈ circles with diameters IA F and IA D respectively. ∵ B1 = midpoint(DF ).
Similarly,
A1 = midpoint(EF ) and C1 = midpoint(DE). So under this inversion,
}ABC −→ nine point circle(DEF ). Now let N be the nine point center(4DEF ).
then by properties of inversion , we have O ∈ IA N . Also note that IA , N, H lie on the euler
line of4DEF .
∵ O ∈ IA H. This proves the claim. 
Now back to the problem , }ABC is fixed. =⇒ O is fixed. By claim, we know IA H always
passes through O.
∵ IA H always passes through a fixed point

Solutions by Aahan Chatterjee Page 2


Math Problem Solutions September 8, 2019

Problem 4
ANS: f (y) = y for all y ∈ N

Let P(x, y)denote the assertion :


x+f (y)2 |xf (x) + 2xf (y)2 + y 4

Claim 1: f(p) = pk for some k ∈ N .

Proof: Let q be a prime dividing f (p).


P (pq, p)
=⇒ pq + f (p)2 |pqf (pq) + 2pqf (p)2 + p4
q|f (p). =⇒ q|pqf (pq) + 2pqf (p)2 + p4 q|pqf (pq) + 2pqf (p)2 . =⇒ q|p4 .
=⇒ q = p. ∵ f (p) = pk

Claim 2: f(1) = 1.

Proof: 1 + f (1)2 |f (1) + 2f (1)2 + 1 =⇒ 1 + f (1)2 |1 − f (1).


Now since f (1) ≥ 1, 1 + f (1)2 > 1 − f (1).
=⇒ f (1) = 1 
. Now back to the problem, P (p, 1)
=⇒ p + 1|p(pk + 1).
=⇒ p+1|(pk +1) [since p, p+1 are co - prime] =⇒ k is odd. Also P (1, p) =⇒ p2k +1|1+2p2k +p4
=⇒ p2k + 1|p4 − 1.
=⇒ k ≤ 2. ∵ k = 1
∵ f (p) = p. Now P (y, 1) =⇒ y + 1|f (y) + 1. =⇒ f (y) ≥ y and P (1, y) =⇒ f (y)2 + 1|y 4 + 1 +
2f (y)2 .
f (y)2 + 1|y 4 − 1. =⇒ f (y) ≤ y 2 . Now let p be any prime and y any natural number, then
P (p, y)
=⇒ p+f (y)2 |p2 +2pf (y)2 +y 4 . =⇒ p+f (y)2 |f (y)4 −y 4 . Now if f (y) 6= y, p+f (y)2 ≤ f (y)4 −y 4
[since y ≤ f (y)]. =⇒ p ≤ y 8 − (y 4 + y 2 ) [using y ≤ f (y) ≤ y 2 ]. =⇒ p has an upper limit for a
fixed y which is impossible since p varies over all primes and there are infinitely many primes.
∵ f (y) = y for all y ∈ N

Solutions by Aahan Chatterjee Page 3


Math Problem Solutions September 8, 2019

Problem 6

A
J

R
Y

M
E
Z
P
I
X F

C
D L

N
H K
B U

Let w denote the incircle of 4ABC.


Claim 1: YZ is tangent to w.
Proof: Let M = DP ∩ w; and let the tangent to w meet AB, AC at Z1 and Y1 respectively.
Now by Newton’s Theorem on quadrilateral BCY1 Z1 , we have BY1 ∩ CZ1 = DM ∩ EF = P
⇒ P ∈ BY1 and P∈ CZ1
∵ Y = Y1 and Z = Z1 . This proves the claim. 
Claim 2:
Let M be any point on }ABC. Let the tangents from X to w intersect w at Y1 , Y2 and
BC extended at X1 , X2 and let the point of tangency between the A-Mixtilinear incircle and
}ABC be U . Then }M X1 X2 passes through U.

Solutions by Aahan Chatterjee Page 4


Math Problem Solutions September 8, 2019

F Y2

Y1
X2
N

X1 C
B
D

Proof: Let N = AM ∩BC By Dual of Desargues Involution Theorem on complete quadrilateral


ABDC athrough M , giving the involutive pairing
(M A, M D); (M B, M C); (M Y1 , M Y2 ). Now projecting this onto line BC, we get the pairs :
(M N, M D); (M B, M C); (M X1 , M X2 ). Now we know that every involution is an inversion
about some center. Let this center be K.
KB × KC = KD × KN = KX1 × KX2 . So }AM BC , }M DN , }M X1 X2 are co- axial
circles. So it suffices to prove U ∈ }M DN .
Now let DU ∩ }ABC=A1 . By properties of mixtilinear incircles, AA1 kBC. ∵ ∠AN B =
∠A1 AM = ∠DU M . So U DN M is cyclic as required. 
Claim 3:
Let XY ∩ }ABC = J. Then the tangents from X and J to w intersect at }ABC.

Proof: Let the tangents from X and J to w meet BC at H, L and let T = Y Z ∩ BC. Now
by claim 1, T X and T H are tangent to w. So by claim 2, }T XH and }T JL pass through U .
=⇒ by Miquel’s Theorem, U is the miquel point of quadrilateral XHLJ.
∵ XH ∩ JL lies on }XU J = }ABC. 

Solutions by Aahan Chatterjee Page 5


Math Problem Solutions September 8, 2019

L K

O
C
D M
B

Claim 4:
AI ∩ RD ∈ }ABC

Proof: LEMMA 1 : T ∈ AR
Firstly we prove BY CZ is cyclic. ∠P ED = 90- B2 =⇒ ∠M DE = B2
=⇒ ∠M Y E = 180-B. So BY CZ is cyclic. First we consider all polars wrt. w.
Polar(A)= EF and Polar(T )=DM .
Since P = EF ∩ DM , Polar(P )= AT .
=⇒ P I ⊥ AT
Now we consider all polars wrt. }BCY Z
By Brocard’s Theorem on }BCY Z, we know Polar(P ) = AT . Let center of }BCY Z = O.
=⇒ P O ⊥ AT . =⇒ R = OP ∩ }ABC.
So by Miquel’s Theorem R is the Miquel point of cyclic quadrilateral BCY Z. Since T =
Y Z ∩ BC, T ∈ AR.

LEMMA 2: Let ABC be a triangle with circumcenter O and let the foot of perpendicular from
A onto BC be D. Then }AOD intersects the perpendicular bisector of BC at the antipode of
D wrt. nine point circle of 4ABC.

Let M be the midpoint of BC, w1 the nine point circle of 4ABC, K1 = OM ∩ w1 and L =
AD ∩ w1 . We have AD ⊥ BC ,OM ⊥ BC and K, L, D, M lieonw1 .
=⇒ KLDM is a rectangle.
So KL = DM .
Also note that L is the midpoint of AH. We know OM = AH 2 .
=⇒ AL = AH = OM . So 4ALK ∼
= 4OM D. =⇒ ∠DOM = ∠KAL.
2
So AKOD is cyclic as desired.

Solutions by Aahan Chatterjee Page 6


Math Problem Solutions September 8, 2019

Now back to the main claim, let us invert about the incircle. We claim R −→ P .
For this notice that ∠IRA = 90 (By LEMMA 1). Suppose image of R is R1 and image of A is
A1 . Then A1 ∈ EF ,∠IA1 E = 90 , IA1 ⊥ A1 R1 and R1 ∈ IP .
R1 = P . So it suffices to prove that }DIP ∩ IA lies on the nine point circle of triangle DEF .
But by LEMMA 2, we know that the perpendicular bisector of EF and }DIP intersect at the
antipode of D wrt. nine point circle of DEF , which proves our claim. 
Now back to the problem, invert about A with r2 = AZ × AB. Let the images of X and J be
X1 and J1 .
Note that Z −→ B, Y −→ C.
=⇒ }ABC −→ Y Z. Since X ∈ }ABC, X1 ∈ Y Z. Also X1 ∈ AX. But X ∈ Y Z.
∵ X1 = X. Similarly J1 = J. So X and J remain fixed.
=⇒ AX = AJ. So A is the midpoint of XJ. ˆ Let XH ∩ JL = K. So I is the incenter of
triangle XJK. Since K lies on }ABC (by claim 3)and A is the midpoint of XJ ˆ , K ∈ AI.
Let K1 = IA ∩ RD. By claim 4, K1 ∈ w.
=⇒ K1 = K. So we are done.

Solutions by Aahan Chatterjee Page 7

You might also like